1. Standard memberChronicLeaky
    Don't Fear Me
    Reaping
    Joined
    28 Feb '07
    Moves
    655
    04 Jun '08 01:53
    Originally posted by FabianFnas
    Well, it was a wild guess, the closest I can get to an answer.

    Is it possible to construct a plane where inf really is included? As you can with R, i.e. R* ?
    Well, you can use the Riemann sphere, although it's not really a "plane", being compact, having different Euler characteristic, etc.
  2. Standard memberwolfgang59
    Quiz Master
    RHP Arms
    Joined
    09 Jun '07
    Moves
    48793
    04 Jun '08 07:23
    'Originally posted by Jirakon
    A point is an ordered pair of two REAL numbers. No infinity.

    If that's the case, then there is no solution:

    Suppose such a set exists in which all points have finite coordinates. Take the point(s) with the highest y-value (y_max). Now imagine the line y = y_max + 1. This line does not intersect any of the points. Therefore no such set exists.
    Just because all points have finite coordinates does not mean there is a 'y-max'

    Otherwise your "proof" could be used to prove that the set of points on x=0 is finite.
  3. Joined
    07 Sep '05
    Moves
    35068
    04 Jun '08 09:081 edit
    Originally posted by Palynka
    I don't think it can be discrete points. Imagine if you have a set of discrete points and pick a line that intersects only two points. If you pick one of those points as center and keep rotating that line infinitesimally, then I don't see how it's possible to cover all possibilities with any mesh of discrete points.
    Well, it would have to be something complicated - discrete but dense. Think of those functions that have different values depending on whether x is rational or irrational. You can get some very strange behaviour.

    I agree, I don't think there is such as set, but I do know that you can construct sets with properties that intuition really doesn't work well on. So I'll need a proper proof to be sure.
  4. Standard memberPalynka
    Upward Spiral
    Halfway
    Joined
    02 Aug '04
    Moves
    8702
    04 Jun '08 09:47
    Originally posted by Jirakon
    A point is an ordered pair of two REAL numbers. No infinity.

    If that's the case, then there is no solution:

    Suppose such a set exists in which all points have finite coordinates. Take the point(s) with the highest y-value (y_max). Now imagine the line y = y_max + 1. This line does not intersect any of the points. Therefore no such set exists.
    I don't see why what I said implies the set must be bounded. Take the Euclidean axis. No line that you can draw or represent as a function intersects a point of those axes only at infinity.
  5. Standard memberPalynka
    Upward Spiral
    Halfway
    Joined
    02 Aug '04
    Moves
    8702
    04 Jun '08 09:54
    Originally posted by mtthw
    Well, it would have to be something complicated - discrete but dense. Think of those functions that have different values depending on whether x is rational or irrational. You can get some very strange behaviour.

    I agree, I don't think there is such as set, but I do know that you can construct sets with properties that intuition really doesn't work well on. So I'll need a proper proof to be sure.
    But if it's discrete, you'd always find a rotation of an infinitesimal epsilon that slips through a discontinuity. Note that every two points in the set define a unique line and all possible lines must be defined by that set.
  6. Joined
    07 Sep '05
    Moves
    35068
    04 Jun '08 10:17
    Originally posted by Palynka
    But if it's discrete, you'd always find a rotation of an infinitesimal epsilon that slips through a discontinuity. Note that every two points in the set define a unique line and all possible lines must be defined by that set.
    OK try S = {x, y: s.t. x and y are irrational}

    Even an infinitesimal rotation is going to sweep through an infinite number of points.

    Yes, I know that that set breaks lots of the other conditions - I'm just using it to illustrate the point. Dense sets - and I mean a set such that any interval is going to contain at least one value - can have some weird properties.
  7. B is for bye bye
    Joined
    09 Apr '06
    Moves
    27526
    04 Jun '08 11:00
    Originally posted by mtthw
    OK try S = {x, y: s.t. x and y are irrational}

    Even an infinitesimal rotation is going to sweep through an infinite number of points.

    Yes, I know that that set breaks lots of the other conditions - I'm just using it to illustrate the point. Dense sets - and I mean a set such that any interval is going to contain at least one value - can have some weird properties.
    But there are still holes in that set. The irrational set is uncountable and therefore would have holes.
  8. Joined
    07 Sep '05
    Moves
    35068
    04 Jun '08 12:381 edit
    Originally posted by Gastel
    But there are still holes in that set. The irrational set is uncountable and therefore would have holes.
    Yes, it would. That's why I called it a discrete set. It's designed to illustrate the problems with one specific objection.

    Oh, and if I was going to be picky 🙂 - it's uncountable and there are holes - but it's not got holes because it's uncountable.
  9. Standard memberPalynka
    Upward Spiral
    Halfway
    Joined
    02 Aug '04
    Moves
    8702
    04 Jun '08 14:25
    Originally posted by mtthw
    OK try S = {x, y: s.t. x and y are irrational}

    Even an infinitesimal rotation is going to sweep through an infinite number of points.

    Yes, I know that that set breaks lots of the other conditions - I'm just using it to illustrate the point. Dense sets - and I mean a set such that any interval is going to contain at least one value - can have some weird properties.
    Nice example. I did add my note that every set of two points in the S set defines a unique line, but I see your point about proving it formally.
  10. Southern California
    Joined
    14 Jun '08
    Moves
    118
    15 Jun '08 03:49
    Math! One of my best subjects... give me a problem!! 😀
  11. Joined
    11 Nov '05
    Moves
    43938
    15 Jun '08 14:081 edit
    Originally posted by Yassy94
    Math! One of my best subjects... give me a problem!! 😀
    Prove, that 1+2=3

    Is this provable? Yes it is.
    Most of math students, from lower grade of university, cannot. Can you?
  12. R
    Standard memberRemoved
    Joined
    10 Dec '06
    Moves
    8528
    15 Jun '08 14:30
    Originally posted by FabianFnas
    Prove, that 1+2=3

    Is this provable? Yes it is.
    Most of math students, from lower grade of university, cannot. Can you?
    by "lower grade" do you mean lower caliber, or lower amount of time attended?

    because I can not prove it....
  13. Joined
    11 Nov '05
    Moves
    43938
    15 Jun '08 14:38
    Originally posted by joe shmo
    by "lower grade" do you mean lower caliber, or lower amount of time attended?

    because I can not prove it....
    No, not at all pejorative.

    From when you enter the math classes in University, you have to wait quite a while before you get the hint why 1+2=3. The reason is that there is so many important things to go through before you go into the very essence of the number systems. You take it for obvious that 1+2=3 and you don't have to care much about it.

    So, who can come up with the water strong proof that 1+2=3?
  14. At the Revolution
    Joined
    15 Sep '07
    Moves
    5073
    15 Jun '08 21:19
    Originally posted by FabianFnas
    No, not at all pejorative.

    From when you enter the math classes in University, you have to wait quite a while before you get the hint why 1+2=3. The reason is that there is so many important things to go through before you go into the very essence of the number systems. You take it for obvious that 1+2=3 and you don't have to care much about it.

    So, who can come up with the water strong proof that 1+2=3?
    It's a postulate, just like the one that goes 1+1=2. We have no conclusive way of proving it, yet it exists. It's a bit like "Between every two points there is only one line." Can we prove it? No. Why not? Because we cannot demonstrate using points or lines because in the real world they do not exist. Same thing with 1+2=3 or 1+1=2. If 1+2=3 is a theorem, then I actually don't know because I'm not out of high school yet.
  15. Joined
    11 Nov '05
    Moves
    43938
    15 Jun '08 21:25
    Originally posted by scherzo
    It's a postulate, just like the one that goes 1+1=2. We have no conclusive way of proving it, yet it exists. It's a bit like "Between every two points there is only one line." Can we prove it? No. Why not? Because we cannot demonstrate using points or lines because in the real world they do not exist. Same thing with 1+2=3 or 1+1=2. If 1+2=3 is a theorem, then I actually don't know because I'm not out of high school yet.
    No no, 1+2=3 is certainly provable. Just use the axioms. The axioms, on the other hand, is not provable. If they were they are not axioms. And now Gödel comes into my mind.
Back to Top

Cookies help us deliver our Services. By using our Services or clicking I agree, you agree to our use of cookies. Learn More.I Agree